GMAT Club
close
Guest online
My chats
Join group
New group
New private
Archived groups
go back
Quant Question of the Day 3 members online out of 3744
dots
Chat members
gmatophobia wrote:
Hey 0Lucky0 Thanks for posting your question. The questions asked on the official exams on P&C are relatively easier than most of the questions discussed on the forum. Hence, don’t get bogged down if you are getting the tough ’non-official’ questions incorrect. Having said that, you can use the principle of counting and filling space method to solve most, if not all, of the official questions. I would recommend that you try to visualize the problem as if you are performing the action. Doing so will help you solve questions with ease and you can use reasoning rather than formulas to ace the question. Here are a couple of great articles by Bunuel and Other experts on P&C that you may want to give read - https://gmatclub.com/forum/combinatoric ... 06266.html https://gmatclub.com/forum/learn-struct ... 63129.html https://gmatclub.com/forum/3-deadly-mis ... 63721.html Set of P&C practice questions by Narren- https://gmatclub.com/forum/practice-que ... 50387.html Feel free to reach out with any additional questions.

Thanks. :) But the thing is, I am only solving the Official questions and haven’t touched any non-official questions yet. :P
So I guess I am in for a tough ride. :P Well, I agree that counting and filling space works but it takes too much time. If I end up spending 5 minutes just on 1 question then I wouldn’t have time for remaining questions. That’s why I wanted to improve my Combinations skills on tough questions somehow. I don’t really have any issues with medium level combinations questions, it’s just the tough ones where there are graphs and diagrams or some other crazy stuff. ie, mixing other skills/concepts into combinations questions. Those are what make my head spin. :P
But, after having thought about this, I think it’s because I haven’t yet mastered those other skills maybe? So that’s why I guess. Not sure.
chevron down
Quote
Private
Delete
Pin
User avatar
gmatophobia wrote:
DS Question 1 - Apr 11 If p, q and r are integers, and pq + r is an odd integer, is p an even integer? (1) pq + pr is an even integer. (2) p + qr is an odd integer. Source: Expert’s Global | Difficulty: Hard

A?
chevron down
Quote
Private
Delete
Pin
User avatar
0Lucky0 wrote:
Thanks. :) But the thing is, I am only solving the Official questions and haven’t touched any non-official questions yet. :P So I guess I am in for a tough ride. :P Well, I agree that counting and filling space works but it takes too much time. If I end up spending 5 minutes just on 1 question then I wouldn’t have time for remaining questions. That’s why I wanted to improve my Combinations skills on tough questions somehow. I don’t really have any issues with medium level combinations questions, it’s just the tough ones where there are graphs and diagrams or some other crazy stuff. ie, mixing other skills/concepts into combinations questions. Those are what make my head spin. :P But, after having thought about this, I think it’s because I haven’t yet mastered those other skills maybe? So that’s why I guess. Not sure.

You can share the question on this group so that someone can help you with the reasoning or their thought process.
chevron down
Quote
Private
Delete
Pin
gmatophobia wrote:
PS Question 1 - Apr 11 How many positive integers less than 10,000 are such that the product of their digits is 30? A. 12 B. 24 C. 36 D. 38 E. 50 Source: GMAT Club Tests | Difficulty: Hard

Ways to create 30:
30*1
15*2
6*5
2*3*5
Now, there is no overlap between the ways to create 30 - so this makes the question easier.
Also note it is the product of their digits - so there cannot be a 0, otherwise it is 0. This excludes 30*1 from the list.
The above also tells us that the numbers may not have any 0s in between them. So something like 1520 is not possible.
that leaves:
ways to arrange 2,3,5=3!=6
ways to arrange 6*5=2!=2
ways to arrange 1,5,2=3!=6
Also consider adding a few 1s:
ways to arrange 1,1,5,2=4!/2!=24/2=12
ways to arrange 1,6,5=3!=6
ways to arrange 1,2,3,5=4!=24
and also
1,1,6,5=4!/2!-12.
For a sum of: 6+2+6+12+6+24+12=68
chevron down
Quote
Private
Delete
Pin
the logic is correct but I am doing something wrong :sleep:
chevron down
Quote
Private
Delete
Pin
gmatophobia wrote:
PS Question 1 - Apr 08 Positive integer z leaves the remainder of 1 when divided by 3, the remainder of 3 when divided by 5 and the remainder of 5 when divided by 7. What is the smallest possible value of z? A. 101 B. 103 C. 105 D. 107 E. 208 Source: GMAT Club Tests | Difficulty: Medium

Easy way to do this:
Eliminate A since it would give remainder 2 when divided by 3 rather than remainder 1.
Eliminate C since that divides evenly into 5.
Eliminate D for the same reasons as A - R2 when divided by 3 rather than R1.
That leaves B and E.
E divided by 5 gives a remainder of 3 when divided by 5, and when divided by 7 it is 5. E matches all the rules.
B: 103/7 gives remainder 5, and 103/5 gives remainder 3.

Since we are looking for the smallest value and both B and E work, it must be B.
chevron down
Quote
Private
Delete
Pin
User avatar
mysterymanrog wrote:
Ways to create 30: 30*1 15*2 6*5 2*3*5 Now, there is no overlap between the ways to create 30 - so this makes the question easier. Also note it is the product of their digits - so there cannot be a 0, otherwise it is 0. This excludes 30*1 from the list. The above also tells us that the numbers may not have any 0s in between them. So something like 1520 is not possible. that leaves: ways to arrange 2,3,5=3!=6 ways to arrange 6*5=2!=2 ways to arrange 1,5,2=3!=6 Also consider adding a few 1s: ways to arrange 1,1,5,2=4!/2!=24/2=12 ways to arrange 1,6,5=3!=6 ways to arrange 1,2,3,5=4!=24 and also 1,1,6,5=4!/2!-12. For a sum of: 6+2+6+12+6+24+12=68

say 2,3,5 and 1,2,3,5 combos will mean the same thing as we are anyway multiplying the digits to make product as 30, so one of the digit has to be "1" in either case

also, i see you are taking 1,1,5,2 as one of the combinations. but note that this will not give the product of all digits as 30

if we remove these cases then final answer should be 68-6-12 = 50
chevron down
Quote
Private
Delete
Pin
User avatar
mysterymanrog wrote:
the logic is correct but I am doing something wrong :sleep:

so your 1,2,3,5 and 2,3,5 are overlapping (because even if not mentioning "1" we ought one of the digit as "1")

also 1,1,5,2 and 1,5,2 are overlapping as well as not a valid case
chevron down
Quote
Private
Delete
Pin
User avatar
but since I now realized :facepalm_man: that you meant 1,2,3,5 and 2,3,5 are four-digit and three-digit numbers separately, then we only ought to remove 1,1,5,2 and 1,5,2 cases ie. a total of 12+6 cases from 68
chevron down
Quote
Private
Delete
Pin
AbhinavKumar wrote:
but since I now realized :facepalm_man: that you meant 1,2,3,5 and 2,3,5 are four-digit and three-digit numbers separately, then we only ought to remove 1,1,5,2 and 1,5,2 cases ie. a total of 12+6 cases from 68

Yeah
chevron down
Quote
Private
Delete
Pin
gmatophobia wrote:
DS Question 1 - Apr 11 If p, q and r are integers, and pq + r is an odd integer, is p an even integer? (1) pq + pr is an even integer. (2) p + qr is an odd integer. Source: Expert’s Global | Difficulty: Hard

A
Q Stem: Pq+r is odd
P/&Q - Even, R -Odd OR
P&Q - Odd, R -Even
Stmt 1: P(Q+R) is Even
Combine with Q stem
P - Even, R - Odd. OR
P- Odd, Q&R - Even or Q&R - Odd; Both contradict Q stem
chevron down
Quote
Private
Delete
Pin
gmatophobia wrote:
PS Question 1 - Apr 11 How many positive integers less than 10,000 are such that the product of their digits is 30? A. 12 B. 24 C. 36 D. 38 E. 50 Source: GMAT Club Tests | Difficulty: Hard

factors possible (5,6) & (2,3,5)
1 digit - Not Possible
2 digits - (5,6) = 2!
3 digits - (2,3,5) = 3!
(1,5,6) = 3!
4 digits - (1,2,3,5) = 4!
(1,1,5,6)=4!/2!
Total=50
chevron down
Quote
Private
Delete
Pin
mysterymanrog wrote:
Ways to create 30: 30*1 15*2 6*5 2*3*5 Now, there is no overlap between the ways to create 30 - so this makes the question easier. Also note it is the product of their digits - so there cannot be a 0, otherwise it is 0. This excludes 30*1 from the list. The above also tells us that the numbers may not have any 0s in between them. So something like 1520 is not possible. that leaves: ways to arrange 2,3,5=3!=6 ways to arrange 6*5=2!=2 ways to arrange 1,5,2=3!=6 Also consider adding a few 1s: ways to arrange 1,1,5,2=4!/2!=24/2=12 ways to arrange 1,6,5=3!=6 ways to arrange 1,2,3,5=4!=24 and also 1,1,6,5=4!/2!-12. For a sum of: 6+2+6+12+6+24+12=68

you can’t have (1,2,5) or (1,1,2,5)
I think you misread the question. product is supposed to be 30
chevron down
Quote
Private
Delete
Pin
wickedvikram wrote:
A Q Stem: Pq+r is odd P/&Q - Even, R -Odd OR P&Q - Odd, R -Even Stmt 1: P(Q+R) is Even Combine with Q stem P - Even, R - Odd. OR P- Odd, Q&R - Even or Q&R - Odd; Both contradict Q stem

Forgot stmt 2 :) :
here, when P is odd, at least one out of Q, R needs to be Even. this is possible without contradicting q stem
also, p can be even with both Q&R odd without contradiction
chevron down
Quote
Private
Delete
Pin
User avatar
mysterymanrog wrote:
the logic is correct but I am doing something wrong :sleep:

I would do this by segregating the three digit and four digit numbers. I believe you’re double counting cases. Well tried ! :thumbsup:
chevron down
Quote
Private
Delete
Pin
User avatar
gmatophobia wrote:
DS Question 1 - Apr 11 If p, q and r are integers, and pq + r is an odd integer, is p an even integer? (1) pq + pr is an even integer. (2) p + qr is an odd integer. Source: Expert’s Global | Difficulty: Hard

chevron down
Quote
Private
Delete
Pin
User avatar
gmatophobia wrote:
PS Question 1 - Apr 11 How many positive integers less than 10,000 are such that the product of their digits is 30? A. 12 B. 24 C. 36 D. 38 E. 50 Source: GMAT Club Tests | Difficulty: Hard

chevron down
Quote
Private
Delete
Pin
User avatar
DS Question 1 - Apr 12

A bookstore bought 24 books, each at an equal price. Each of the first 20 books was sold at the same price and each of the remaining 4 books was sold at a 50% higher price. Was the profit on the sale of 24 books greater than $240?

(A) The profit on each of the 20 books was $10.

(B) If the selling price had been $4 more per book for all 24 books, the overall profit would have been $376.

Source: GMATWhiz | Difficulty: Hard
chevron down
Quote
Private
Delete
Pin
User avatar
PS Question 1 - Apr 12

The median of 5 numbers is 50, and their range is 40. If the median of the 3 smallest numbers is 40, which of the following could be the range of the 3 largest numbers?

I. 0
II. 20
III. 40

A. I only
B. II only
C. I & II only
D.I & III only
E. I, II & III

Source: Math Revolution | Difficulty: Medium
chevron down
Quote
Private
Delete
Pin
gmatophobia wrote:
DS Question 1 - Apr 12 A bookstore bought 24 books, each at an equal price. Each of the first 20 books was sold at the same price and each of the remaining 4 books was sold at a 50% higher price. Was the profit on the sale of 24 books greater than $240? (A) The profit on each of the 20 books was $10. (B) If the selling price had been $4 more per book for all 24 books, the overall profit would have been $376. Source: GMATWhiz | Difficulty: Hard

D
chevron down
Quote
Private
Delete
Pin
gmatophobia wrote:
PS Question 1 - Apr 12 The median of 5 numbers is 50, and their range is 40. If the median of the 3 smallest numbers is 40, which of the following could be the range of the 3 largest numbers? I. 0 II. 20 III. 40 A. I only B. II only C. I & II only D.I & III only E. I, II & III Source: Math Revolution | Difficulty: Medium

C
chevron down
Quote
Private
Delete
Pin
User avatar
gmatophobia wrote:
DS Question 1 - Apr 12 A bookstore bought 24 books, each at an equal price. Each of the first 20 books was sold at the same price and each of the remaining 4 books was sold at a 50% higher price. Was the profit on the sale of 24 books greater than $240? (A) The profit on each of the 20 books was $10. (B) If the selling price had been $4 more per book for all 24 books, the overall profit would have been $376. Source: GMATWhiz | Difficulty: Hard

either B or D. lets go with D
chevron down
Quote
Private
Delete
Pin
User avatar
gmatophobia wrote:
PS Question 1 - Apr 12 The median of 5 numbers is 50, and their range is 40. If the median of the 3 smallest numbers is 40, which of the following could be the range of the 3 largest numbers? I. 0 II. 20 III. 40 A. I only B. II only C. I & II only D.I & III only E. I, II & III Source: Math Revolution | Difficulty: Medium

C
chevron down
Quote
Private
Delete
Pin
wickedvikram wrote:
D

let CP be x and SP be y for each book
for 24, 24x & 20y+4*3y/2=26y
26y-24x>240
stmt1: y-x =10
24(y-x)=240
hence 2y+240 is greater than 240
stmt2: 20(y+4)+4*3/2*(y+4)=26y+104
26y+104-24x=376
26y-24x=272
chevron down
Quote
Private
Delete
Pin
wickedvikram wrote:
C

10, 40, 50, 50, 50 : Range 0
30, 40, 50, 50, 70 : Range 20
range 40 is not possible since 2nd element is smaller than 3rd element and range of 5 elements 50
chevron down
Quote
Private
Delete
Pin
***range of 5 elements 40
chevron down
Quote
Private
Delete
Pin
wickedvikram wrote:
let CP be x and SP be y for each book for 24, 24x & 20y+4*3y/2=26y 26y-24x>240 stmt1: y-x =10 24(y-x)=240 hence 2y+240 is greater than 240 stmt2: 20(y+4)+4*3/2*(y+4)=26y+104 26y+104-24x=376 26y-24x=272

hey could you please explain why you put 24 (y-x) instead of 20 (y-x) as the statement suggests that the profit on each of the 20 books was 20.
chevron down
Quote
Private
Delete
Pin
hey could you please explain why you put 24 (y-x) instead of 20 (y-x) as the statement suggests that the profit on each of the 20 books was *10*, sorry
chevron down
Quote
Private
Delete
Pin
blackswan33 wrote:
hey could you please explain why you put 24 (y-x) instead of 20 (y-x) as the statement suggests that the profit on each of the 20 books was *10*, sorry

I solved the total SP for 20 & 4 books together
i.e. 20(books) y(SP for 20)+4(books)*3y/2(SP for 4)=26y
I need, 26y-24x > 240
statement 1 only gives me y-x,
26y-24x= 2y+24y-24x=2y+24*10=2y+240
since y (SP) can’t be less than 10 (profit amount), I am getting a number above 260
chevron down
Quote
Private
Delete
Pin
Also, when the CP is same for all 24 books and SP is at a profit of $10 for 20 books. Profit of remaining 4 books will be higher as SP 1.5 times
chevron down
Quote
Private
Delete
Pin
User avatar
gmatophobia wrote:
DS Question 1 - Apr 12 A bookstore bought 24 books, each at an equal price. Each of the first 20 books was sold at the same price and each of the remaining 4 books was sold at a 50% higher price. Was the profit on the sale of 24 books greater than $240? (A) The profit on each of the 20 books was $10. (B) If the selling price had been $4 more per book for all 24 books, the overall profit would have been $376. Source: GMATWhiz | Difficulty: Hard

chevron down
Quote
Private
Delete
Pin
User avatar
gmatophobia wrote:
PS Question 1 - Apr 12 The median of 5 numbers is 50, and their range is 40. If the median of the 3 smallest numbers is 40, which of the following could be the range of the 3 largest numbers? I. 0 II. 20 III. 40 A. I only B. II only C. I & II only D.I & III only E. I, II & III Source: Math Revolution | Difficulty: Medium

chevron down
Quote
Private
Delete
Pin
User avatar
DS Question 1 - Apr 13

If y=|x|+x, where y is an integer. Is y = 0 ?

(1) x!+7 is prime

(2) y < 2

Source: Others | Difficulty: Hard
chevron down
Quote
Private
Delete
Pin
User avatar
PS Question 1 - Apr 13

Pam and Robin each roll a pair of fair, six-sided dice. What is the probability that Pam and Robin will both roll the same set of two numbers?

A. 1/216
B. 1/36
C. 5/108
D. 11/216
E. 1/18

Source: Veritas Prep | Difficulty: Hard
chevron down
Quote
Private
Delete
Pin
User avatar
gmatophobia wrote:
DS Question 1 - Apr 13 If y=|x|+x, where y is an integer. Is y = 0 ? (1) x!+7 is prime (2) y < 2 Source: Others | Difficulty: Hard

if y=|x|+x
is y = 0 ?
it is possible only if x is negative

Statement 1:

x!+7 = prime
and that prime can be 2,3,5,7,11,13, so on

BUT

x! = -5,-4,-4,0,4 for primes 2 to 11
so min value of prime for which x!+7 = prime = 13
or x! = 6
or x = 3

therefore x >=3 (always positive) hence y != 0

Statement 2:

y<2 then y can be 0 or (say) 1, so not a sufficient statement


hence going for option A
chevron down
Quote
Private
Delete
Pin
User avatar
(edit )***
BUT
x! = -5,-4,-2,0,4 for primes 2 to 11 s
(such values of x! which is not possible)

so min value of prime for which x!+7 = prime = 13
or x! = 6
or x = 3
chevron down
Quote
Private
Delete
Pin
gmatophobia wrote:
PS Question 1 - Apr 13 Pam and Robin each roll a pair of fair, six-sided dice. What is the probability that Pam and Robin will both roll the same set of two numbers? A. 1/216 B. 1/36 C. 5/108 D. 11/216 E. 1/18 Source: Veritas Prep | Difficulty: Hard

D - 11/216
chevron down
Quote
Private
Delete
Pin
wickedvikram wrote:
D - 11/216

Solution: I attempted this with an imaginary 2 face dice and extrapolated all the scenarios for 6 faced dice
Case 1: All numbers are same 11-11,22-22,33-33,.....66-66 = 6 cases
Case 2: Here, there is a small catch, 12 for pam can be 12 &21 for robin. Why? because it’s a selection not arrangement between the two.
So, for one number 5*2 cases (we already counted same numbers)
for 6 numbers = 6*5*2=60 cases
total = 66
we already know for four dices number of cases are 6*6*6*6
Probability 66/6*6*6*6 =11/216
chevron down
Quote
Private
Delete
Pin
gmatophobia wrote:
DS Question 1 - Apr 13 If y=|x|+x, where y is an integer. Is y = 0 ? (1) x!+7 is prime (2) y < 2 Source: Others | Difficulty: Hard

stmt 1: x!+7 is prime; By this statement x is positive integer, since factorial of -ve number is undefined.
also, x is not zero, coz x!+1 will be 8, which is not prime
y can have multiple values, but the all have to be even positive integers
Answer-No; Sufficient
stmt 2: y < 2
now y can take either of 0 or 1, since y is an integer and any negative value of x will fetch a zero
as it is never mentioned that x is an integer, it can be 0.5 for y to be 1
Insufficient

Ans - A
chevron down
Quote
Private
Delete
Pin
Hi!

Actually, I’m new to the forum. Can someone please guide me to all the questions/problem sets from the GMAT Official Guide Verbal & Quant Review 2022?

TIA!!
chevron down
Quote
Private
Delete
Pin
hey! I am new to to this forum, I need some guidance on how to prepare for the GMAT.
chevron down
Quote
Private
Delete
Pin
User avatar
Hi. Welcome to GMAT club. You can start here: https://gmatclub.com/forum/gmat-study-p ... 17827.html
chevron down
Quote
Private
Delete
Pin
wickedvikram wrote:
factors possible (5,6) & (2,3,5) 1 digit - Not Possible 2 digits - (5,6) = 2! 3 digits - (2,3,5) = 3! (1,5,6) = 3! 4 digits - (1,2,3,5) = 4! (1,1,5,6)=4!/2! Total=50

Hi. Why did u divide the (1,1,5,6) option by 2! ?
chevron down
Quote
Private
Delete
Pin
PaulElHajj wrote:
Hi. Why did u divide the (1,1,5,6) option by 2! ?

Because 1 is coming twice. See it as arrangement for words POLE vs POLL
for POLE, number of arrangement for all the alphabets is 4! ; all alphabets are distinct
for POLL, it will be 4!/2!, L is repeating twice in each arrangement.
chevron down
Quote
Private
Delete
Pin
User avatar
gmatophobia wrote:
DS Question 1 - Apr 13 If y=|x|+x, where y is an integer. Is y = 0 ? (1) x!+7 is prime (2) y < 2 Source: Others | Difficulty: Hard

chevron down
Quote
Private
Delete
Pin
User avatar
gmatophobia wrote:
PS Question 1 - Apr 13 Pam and Robin each roll a pair of fair, six-sided dice. What is the probability that Pam and Robin will both roll the same set of two numbers? A. 1/216 B. 1/36 C. 5/108 D. 11/216 E. 1/18 Source: Veritas Prep | Difficulty: Hard

chevron down
Quote
Private
Delete
Pin
User avatar
DS Question 1 - Apr 15

Is ABCD a rectangle?

(1) ∠A = 90º and ∠C = 90º
(2) AD||BC

Source: Experts Global | Difficulty: Hard
chevron down
Quote
Private
Delete
Pin
User avatar
PS Question 1 - Apr 15

Anthony and Michael sit on the six member board of directors for company X. If the board is to be split up into 2 three-person sub-committees, what percent of all the possible subcommittees that include Michael also include Anthony?

(A) 20%
(B) 30%
(C) 40%
(D) 50%
(E) 60%

Source: Manhattan | Difficulty: Hard
chevron down
Quote
Private
Delete
Pin
gmatophobia wrote:
DS Question 1 - Apr 15 Is ABCD a rectangle? (1) ∠A = 90º and ∠C = 90º (2) AD||BC Source: Experts Global | Difficulty: Hard

B
chevron down
Quote
Private
Delete
Pin
gmatophobia wrote:
DS Question 1 - Apr 15 Is ABCD a rectangle? (1) ∠A = 90º and ∠C = 90º (2) AD||BC Source: Experts Global | Difficulty: Hard

C
chevron down
Quote
Private
Delete
Pin
PaulElHajj wrote:
B

C
chevron down
Quote
Private
Delete
Pin
gmatophobia wrote:
PS Question 1 - Apr 15 Anthony and Michael sit on the six member board of directors for company X. If the board is to be split up into 2 three-person sub-committees, what percent of all the possible subcommittees that include Michael also include Anthony? (A) 20% (B) 30% (C) 40% (D) 50% (E) 60% Source: Manhattan | Difficulty: Hard

I found C
chevron down
Quote
Private
Delete
Pin
gmatophobia wrote:
PS Question 1 - Apr 15 Anthony and Michael sit on the six member board of directors for company X. If the board is to be split up into 2 three-person sub-committees, what percent of all the possible subcommittees that include Michael also include Anthony? (A) 20% (B) 30% (C) 40% (D) 50% (E) 60% Source: Manhattan | Difficulty: Hard

total 4 posibilities for 2 sub committees n 2 possibilites for anthony n michael to be in same sub-committee. hence 2/4 is the probability wiz. 50%.
chevron down
Quote
Private
Delete
Pin
gmatophobia wrote:
PS Question 1 - Apr 15 Anthony and Michael sit on the six member board of directors for company X. If the board is to be split up into 2 three-person sub-committees, what percent of all the possible subcommittees that include Michael also include Anthony? (A) 20% (B) 30% (C) 40% (D) 50% (E) 60% Source: Manhattan | Difficulty: Hard

40% should be it.
chevron down
Quote
Private
Delete
Pin
hi
chevron down
Quote
Private
Delete
Pin
gmatophobia wrote:
PS Question 1 - Apr 15 Anthony and Michael sit on the six member board of directors for company X. If the board is to be split up into 2 three-person sub-committees, what percent of all the possible subcommittees that include Michael also include Anthony? (A) 20% (B) 30% (C) 40% (D) 50% (E) 60% Source: Manhattan | Difficulty: Hard

Number of committee possible : 6C3 * 3C3/2! = 10; (Order not needed, so use /2!)
Number of committee with Michael : 10; because there are two committees of 3 from 6
Number of committee with both M&A: 4C1 * 3C3 = 4; (6-2); (first group has M&A, order included, so no /2!)
probability=4/10
Ans - C
chevron down
Quote
Private
Delete
Pin
Hey guys, found a very good question. Give it a try.

If a real number x is randomly chosen between 0 and 2, inclusive, and a real number y is randomly chosen between 0 and 6, inclusive, what is the probability that x>y?

A) 1/12
B) 1/8
C) 1/6
D) 1/4
E) 1/3
chevron down
Quote
Private
Delete
Pin
User avatar
wickedvikram wrote:
Hey guys, found a very good question. Give it a try. If a real number x is randomly chosen between 0 and 2, inclusive, and a real number y is randomly chosen between 0 and 6, inclusive, what is the probability that x>y? A) 1/12 B) 1/8 C) 1/6 D) 1/4 E) 1/3

Nice Ques !

Is the answer C ?

We can use the concept of favorable region / total region
chevron down
Quote
Private
Delete
Pin
gmatophobia wrote:
Nice Ques ! Is the answer C ? We can use the concept of favorable region / total region

Yes, exactly. I got stumped when I saw this question. It has very simple trap that can be missed :)
chevron down
Quote
Private
Delete
Pin
User avatar
gmatophobia wrote:
DS Question 1 - Apr 15 Is ABCD a rectangle? (1) ∠A = 90º and ∠C = 90º (2) AD||BC Source: Experts Global | Difficulty: Hard

chevron down
Quote
Private
Delete
Pin
User avatar
gmatophobia wrote:
PS Question 1 - Apr 15 Anthony and Michael sit on the six member board of directors for company X. If the board is to be split up into 2 three-person sub-committees, what percent of all the possible subcommittees that include Michael also include Anthony? (A) 20% (B) 30% (C) 40% (D) 50% (E) 60% Source: Manhattan | Difficulty: Hard

chevron down
Quote
Private
Delete
Pin
User avatar
DS Question 1 - Apr 17

A certain list consists of 3 different numbers. Does the median of the 3 numbers equal the average (arithmetic mean) of the 3 numbers?

(1) The range of the 3 numbers is equal to twice the difference between the greatest number and the median.
(2) The sum of the 3 numbers is equal to 3 times one of the numbers.

Source: Official Guide | Difficulty: Medium
chevron down
Quote
Private
Delete
Pin
User avatar
PS Question 1 - Apr 17

A washing machine takes 35 minutes to wash one load of laundry, and in between washing different loads of laundry it takes Derek 2 minutes to unload and another 4 minutes to reload the machine. If the washing machine begins washing one load of laundry at 12:30pm, how many loads of laundry can Derek wash and unload before 6:35pm?

(A) 8
(B) 9
(C) 10
(D) 14
(E) 15

Source: Other | Difficulty: Medium
chevron down
Quote
Private
Delete
Pin
gmatophobia wrote:
PS Question 1 - Apr 17 A washing machine takes 35 minutes to wash one load of laundry, and in between washing different loads of laundry it takes Derek 2 minutes to unload and another 4 minutes to reload the machine. If the washing machine begins washing one load of laundry at 12:30pm, how many loads of laundry can Derek wash and unload before 6:35pm? (A) 8 (B) 9 (C) 10 (D) 14 (E) 15 Source: Other | Difficulty: Medium

B
chevron down
Quote
Private
Delete
Pin
gmatophobia wrote:
DS Question 1 - Apr 17 A certain list consists of 3 different numbers. Does the median of the 3 numbers equal the average (arithmetic mean) of the 3 numbers? (1) The range of the 3 numbers is equal to twice the difference between the greatest number and the median. (2) The sum of the 3 numbers is equal to 3 times one of the numbers. Source: Official Guide | Difficulty: Medium

D
a<b<c, b=(a+c)/2 ?
stmt 1: c-a = 2(c-b) ; solve b=(a+c)/2; sufficient
stmt 2: a+b+c=3a or 3b or 3c; i.e. (a+b+c)/3 = a or b or c ;
a & c not possible since average of 3 numbers can’t be the highest or lowest of them. Thus, b remains
a+b+c=3b
a+c=2b; sufficient
chevron down
Quote
Private
Delete
Pin
gmatophobia wrote:
PS Question 1 - Apr 17 A washing machine takes 35 minutes to wash one load of laundry, and in between washing different loads of laundry it takes Derek 2 minutes to unload and another 4 minutes to reload the machine. If the washing machine begins washing one load of laundry at 12:30pm, how many loads of laundry can Derek wash and unload before 6:35pm? (A) 8 (B) 9 (C) 10 (D) 14 (E) 15 Source: Other | Difficulty: Medium

L, W, UL - 4, 35, 2
Time for a cycle until unload = 41 mins
Total time = 365 mins;
wash & unload the first load=365-35-2=328
remaining cycles = 328/41=8
8+1=9
B
chevron down
Quote
Private
Delete
Pin
User avatar
gmatophobia wrote:
DS Question 1 - Apr 17 A certain list consists of 3 different numbers. Does the median of the 3 numbers equal the average (arithmetic mean) of the 3 numbers? (1) The range of the 3 numbers is equal to twice the difference between the greatest number and the median. (2) The sum of the 3 numbers is equal to 3 times one of the numbers. Source: Official Guide | Difficulty: Medium

chevron down
Quote
Private
Delete
Pin
User avatar
gmatophobia wrote:
PS Question 1 - Apr 17 A washing machine takes 35 minutes to wash one load of laundry, and in between washing different loads of laundry it takes Derek 2 minutes to unload and another 4 minutes to reload the machine. If the washing machine begins washing one load of laundry at 12:30pm, how many loads of laundry can Derek wash and unload before 6:35pm? (A) 8 (B) 9 (C) 10 (D) 14 (E) 15 Source: Other | Difficulty: Medium

chevron down
Quote
Private
Delete
Pin
User avatar
DS Question 1 - Apr 18

What is the value of x ?

1) |x| = -x

2) |x|^2 = -x^2

Source: Others | Difficulty: Medium
chevron down
Quote
Private
Delete
Pin
User avatar
PS Question 1 - Apr 18

A certain bakery ran a promotion code: a customer can buy x donuts for the regular price of $15 total and get 3 donuts free. If the donut price per dozen during the promotion is $2 less than the normal donut price per dozen, what is x?

A. 15
B. 18
C. 21
D. 25
E. 30

Source: Manhattan | Difficulty: Hard
chevron down
Quote
Private
Delete
Pin
PS Q1: A
chevron down
Quote
Private
Delete
Pin
gmatophobia wrote:
DS Question 1 - Apr 18 What is the value of x ? 1) |x| = -x 2) |x|^2 = -x^2 Source: Others | Difficulty: Medium

B
chevron down
Quote
Private
Delete
Pin
gmatophobia wrote:
PS Question 1 - Apr 18 A certain bakery ran a promotion code: a customer can buy x donuts for the regular price of $15 total and get 3 donuts free. If the donut price per dozen during the promotion is $2 less than the normal donut price per dozen, what is x? A. 15 B. 18 C. 21 D. 25 E. 30 Source: Manhattan | Difficulty: Hard

A
chevron down
Quote
Private
Delete
Pin
gmatophobia wrote:
PS Question 1 - Apr 18 A certain bakery ran a promotion code: a customer can buy x donuts for the regular price of $15 total and get 3 donuts free. If the donut price per dozen during the promotion is $2 less than the normal donut price per dozen, what is x? A. 15 B. 18 C. 21 D. 25 E. 30 Source: Manhattan | Difficulty: Hard

A
chevron down
Quote
Private
Delete
Pin
gmatophobia wrote:
DS Question 1 - Apr 18 What is the value of x ? 1) |x| = -x 2) |x|^2 = -x^2 Source: Others | Difficulty: Medium

b
chevron down
Quote
Private
Delete
Pin
gmatophobia wrote:
PS Question 1 - Apr 18 A certain bakery ran a promotion code: a customer can buy x donuts for the regular price of $15 total and get 3 donuts free. If the donut price per dozen during the promotion is $2 less than the normal donut price per dozen, what is x? A. 15 B. 18 C. 21 D. 25 E. 30 Source: Manhattan | Difficulty: Hard

A
chevron down
Quote
Private
Delete
Pin
gmatophobia wrote:
DS Question 1 - Apr 18 What is the value of x ? 1) |x| = -x 2) |x|^2 = -x^2 Source: Others | Difficulty: Medium

B
chevron down
Quote
Private
Delete
Pin
User avatar
gmatophobia wrote:
DS Question 1 - Apr 18 What is the value of x ? 1) |x| = -x 2) |x|^2 = -x^2 Source: Others | Difficulty: Medium

chevron down
Quote
Private
Delete
Pin
User avatar
gmatophobia wrote:
PS Question 1 - Apr 18 A certain bakery ran a promotion code: a customer can buy x donuts for the regular price of $15 total and get 3 donuts free. If the donut price per dozen during the promotion is $2 less than the normal donut price per dozen, what is x? A. 15 B. 18 C. 21 D. 25 E. 30 Source: Manhattan | Difficulty: Hard

chevron down
Quote
Private
Delete
Pin
User avatar
DS Question 1 - April 19

Triangle ABC is inscribed in a rectangle ABEF forming two right triangles: AFC and BEC. Is triangle ABC an equilateral triangle?

(1) BE = (3√AB)/2

(2) Point C is the midpoint of EF

Source: eGMAT | Difficulty: Hard
chevron down
Quote
Private
Delete
Pin
User avatar
PS Question 1 - April 19

A lab has 5 Nitrogen cylinders. The standard deviation of of the volume of Nitrogen in the 5 cylinders is 7 gallons. During an experiment, 20% nitrogen was removed from each cylinder; thereafter 2 gallons of nitrogen was added into each cylinder; finally 20% of the then volume in each cylinder, nitrogen was pumped into each cylinder. At the end of 3 steps of the experiment, what was the standard deviation of the volume of Nitrogen in 5 cylinders?

A. 6.72
B. 7
C. 8.72
D. 9.12
E. 11.72

Source: Expert’s Global | Difficulty: Hard (Actually Medium)
chevron down
Quote
Private
Delete
Pin
gmatophobia wrote:
PS Question 1 - April 19 A lab has 5 Nitrogen cylinders. The standard deviation of of the volume of Nitrogen in the 5 cylinders is 7 gallons. During an experiment, 20% nitrogen was removed from each cylinder; thereafter 2 gallons of nitrogen was added into each cylinder; finally 20% of the then volume in each cylinder, nitrogen was pumped into each cylinder. At the end of 3 steps of the experiment, what was the standard deviation of the volume of Nitrogen in 5 cylinders? A. 6.72 B. 7 C. 8.72 D. 9.12 E. 11.72 Source: Expert’s Global | Difficulty: Hard (Actually Medium)

A
chevron down
Quote
Private
Delete
Pin
gmatophobia wrote:
PS Question 1 - April 19 A lab has 5 Nitrogen cylinders. The standard deviation of of the volume of Nitrogen in the 5 cylinders is 7 gallons. During an experiment, 20% nitrogen was removed from each cylinder; thereafter 2 gallons of nitrogen was added into each cylinder; finally 20% of the then volume in each cylinder, nitrogen was pumped into each cylinder. At the end of 3 steps of the experiment, what was the standard deviation of the volume of Nitrogen in 5 cylinders? A. 6.72 B. 7 C. 8.72 D. 9.12 E. 11.72 Source: Expert’s Global | Difficulty: Hard (Actually Medium)

A
chevron down
Quote
Private
Delete
Pin
D
chevron down
Quote
Private
Delete
Pin
gmatophobia wrote:
DS Question 1 - April 19 Triangle ABC is inscribed in a rectangle ABEF forming two right triangles: AFC and BEC. Is triangle ABC an equilateral triangle? (1) BE = (3√AB)/2 (2) Point C is the midpoint of EF Source: eGMAT | Difficulty: Hard

C
chevron down
Quote
Private
Delete
Pin
gmatophobia wrote:
PS Question 1 - April 19 A lab has 5 Nitrogen cylinders. The standard deviation of of the volume of Nitrogen in the 5 cylinders is 7 gallons. During an experiment, 20% nitrogen was removed from each cylinder; thereafter 2 gallons of nitrogen was added into each cylinder; finally 20% of the then volume in each cylinder, nitrogen was pumped into each cylinder. At the end of 3 steps of the experiment, what was the standard deviation of the volume of Nitrogen in 5 cylinders? A. 6.72 B. 7 C. 8.72 D. 9.12 E. 11.72 Source: Expert’s Global | Difficulty: Hard (Actually Medium)

A
chevron down
Quote
Private
Delete
Pin
A
chevron down
Quote
Private
Delete
Pin
PaulElHajj wrote:
C

Is there an option C as well. I can only see 2 two options,A and B. I’m new to this app, so I’m just figuring out.
chevron down
Quote
Private
Delete
Pin
User avatar
SanskritiS wrote:
Is there an option C as well. I can only see 2 two options,A and B. I’m new to this app, so I’m just figuring out.

SanskritiS - Welcome to GMAT Club!

The question you’re referring to a data sufficiency question and the answer choices always consist of the same five options in the same order

A. Statement (1) ALONE is sufficient, but Statement (2) alone is not sufficient to answer the question asked.
B. Statement (2) ALONE is sufficient, but Statement (1) alone is not sufficient to answer the question asked.
C. Both Statements (1) and (2) TOGETHER are sufficient to answer the question asked; but NEITHER statement ALONE is sufficient.
D. EACH statement ALONE is sufficient to answer the question.
E. Statements (1) and (2) TOGETHER are NOT sufficient to answer the question asked, and additional data specific to the problem are needed.

You can read more about data sufficiency question type in this article written by one of the prep companies

https://magoosh.com/gmat/gmat-data-sufficiency-tips/
chevron down
Quote
Private
Delete
Pin
User avatar
gmatophobia wrote:
DS Question 1 - April 19 Triangle ABC is inscribed in a rectangle ABEF forming two right triangles: AFC and BEC. Is triangle ABC an equilateral triangle? (1) BE = (3√AB)/2 (2) Point C is the midpoint of EF Source: eGMAT | Difficulty: Hard

chevron down
Quote
Private
Delete
Pin
User avatar
gmatophobia wrote:
PS Question 1 - April 19 A lab has 5 Nitrogen cylinders. The standard deviation of of the volume of Nitrogen in the 5 cylinders is 7 gallons. During an experiment, 20% nitrogen was removed from each cylinder; thereafter 2 gallons of nitrogen was added into each cylinder; finally 20% of the then volume in each cylinder, nitrogen was pumped into each cylinder. At the end of 3 steps of the experiment, what was the standard deviation of the volume of Nitrogen in 5 cylinders? A. 6.72 B. 7 C. 8.72 D. 9.12 E. 11.72 Source: Expert’s Global | Difficulty: Hard (Actually Medium)

chevron down
Quote
Private
Delete
Pin
User avatar
DS Question 1 - April 21

A music festival took place over two days, Saturday and Sunday. What percent of attendees only attended on Saturday?

(1) 40% of attendees attended both days.

(2) 70% of attendees attended on Sunday.

Source: Vertias Prep | Difficulty: Hard
chevron down
Quote
Private
Delete
Pin
User avatar
PS Question 1 - April 21

If |-x/3 + 1| < 2, which of the following must be true?

A. x > 0
B. x < 8
C. x > -4
D. 0 < x < 3
E. None of the above

Source: Other (Jamboree) | Difficulty: Hard
chevron down
Quote
Private
Delete
Pin
gmatophobia wrote:
SanskritiS - Welcome to GMAT Club! The question you’re referring to a data sufficiency question and the answer choices always consist of the same five options in the same order A. Statement (1) ALONE is sufficient, but Statement (2) alone is not sufficient to answer the question asked. B. Statement (2) ALONE is sufficient, but Statement (1) alone is not sufficient to answer the question asked. C. Both Statements (1) and (2) TOGETHER are sufficient to answer the question asked; but NEITHER statement ALONE is sufficient. D. EACH statement ALONE is sufficient to answer the question. E. Statements (1) and (2) TOGETHER are NOT sufficient to answer the question asked, and additional data specific to the problem are needed. You can read more about data sufficiency question type in this article written by one of the prep companies https://magoosh.com/gmat/gmat-data-sufficiency-tips/

Thank you so much. I didn’t know.
chevron down
Quote
Private
Delete
Pin
User avatar
gmatophobia wrote:
PS Question 1 - April 21 If |-x/3 + 1| < 2, which of the following must be true? A. x > 0 B. x < 8 C. x > -4 D. 0 < x < 3 E. None of the above Source: Other (Jamboree) | Difficulty: Hard

The simplification of equation in question results in:
-3<x<9

although it overshoots the required range, Option C covers the entire range as asked in the question.
So going with option C ?
chevron down
Quote
Private
Delete
Pin
gmatophobia wrote:
DS Question 1 - April 21 A music festival took place over two days, Saturday and Sunday. What percent of attendees only attended on Saturday? (1) 40% of attendees attended both days. (2) 70% of attendees attended on Sunday. Source: Vertias Prep | Difficulty: Hard

B
chevron down
Quote
Private
Delete
Pin
User avatar
Karan0912 wrote:
B

I know it’s unimportant to calculate values in DS, but what percentage of people attended the show on Saturday ? Just want to verify if I’m getting the right answer...
chevron down
Quote
Private
Delete
Pin
User avatar
AbhinavKumar wrote:
I know it’s unimportant to calculate values in DS, but what percentage of people attended the show on Saturday ? Just want to verify if I’m getting the right answer...

30 :)
chevron down
Quote
Private
Delete
Pin
User avatar
AbhinavKumar - Realized that , I misread your question

If your question is "What percentage of people attended the show ONLY on Saturday", the answer is 30 (this is what I have responded to)

If your question is "What percentage of people attended the show on Saturday" , :dontknow:
chevron down
Quote
Private
Delete
Pin
User avatar
Thanks, I asked about "What percentage of people attended the show ONLY on Saturday". it is indeed 30 percent
chevron down
Quote
Private
Delete
Pin